Administración     

Olimpiadas de Matemáticas
Página de preparación y problemas

Selector
La base de datos contiene 1154 problemas y 775 soluciones.
OME Local
OME Nacional
OIM
OME Andalucía
Retos UJA
Problema 315
Sea $p(x)$ un polinomio con coeficientes reales tal que $P(x)>0$ para todo $x\geq0$. Probar que existe un entero positivo $n$ tal que $(1+x)^np(x)$ es un polinomio con todos los coeficientes mayores o iguales que cero.
Sin pistas
Sin soluciones
info
Si crees que el enunciado contiene un error o imprecisión o bien crees que la información sobre la procedencia del problema es incorrecta, puedes notificarlo usando los siguientes botones:
Informar de error en enunciado Informar de procedencia del problema
Problema 283
Sean $\alpha$, $\beta$ y $\gamma$ las raíces del polinomio \[p(x)=x^3-3x+1.\] Hallar otro polinomio de grado $3$ cuyas raíces sean $\frac{1-\alpha}{1+\alpha}$, $\frac{1-\beta}{1+\beta}$ y $\frac{1-\gamma}{1+\gamma}$.
pistasolución 1info
Pista. ¿Qué relación hay entre el polinomio que buscas y $p\left(\frac{1-x}{1+x}\right)$?
Solución. En primer lugar, observamos que $p(-1)\neq 0$, luego los números $\frac{1-\alpha}{1+\alpha}$, $\frac{1-\beta}{1+\beta}$ y $\frac{1-\gamma}{1+\gamma}$ están bien definidos. Consideremos el cambio de variable $y=\frac{1-x}{1+x}$, cuyo cambio inverso es él mismo, es decir, puede despejarse $x=\frac{1-y}{1+y}$. Por tanto, si consideramos \[p\left(\frac{1-y}{1+y}\right)=\left(\frac{1-y}{1+y}\right)^3-3\left(\frac{1-y}{1+y}\right)+1=\frac{3y^3+9y^2-3y-1}{(1+y)^3},\] el miembro de la izquierda es igual a cero para $y=\frac{1-\alpha}{1+\alpha}$, $y=\frac{1-\beta}{1+\beta}$ o $y=\frac{1-\gamma}{1+\gamma}$. Esto quiere decir que el numerador $3y^3+9y^2-3y-1$ también debe ser cero para estos tres valores de $y$ y el polinomio que buscamos es $q(x)=3x^3+9x^2-3x-1$.

Nota. Otra técnica para resolver este problema es usar la ecuaciones de Cardano, pues el polinomio buscado $q(x)=x^3+bx^2+cx+d$ debe cumplir que \begin{eqnarray} b&=&-\frac{1-\alpha}{1+\alpha}-\frac{1-\beta}{1+\beta}-\frac{1-\gamma}{1+\gamma},\\ c&=&\frac{1-\alpha}{1+\alpha}\cdot\frac{1-\beta}{1+\beta}+\frac{1-\alpha}{1+\alpha}\cdot\frac{1-\gamma}{1+\gamma}+\frac{1-\beta}{1+\beta}\cdot\frac{1-\gamma}{1+\gamma},\\ d&=&\frac{1-\alpha}{1+\alpha}\cdot\frac{1-\beta}{1+\beta}\cdot\frac{1-\gamma}{1+\gamma}. \end{eqnarray} Los miembros de la izquierda de estas tres relaciones pueden calcularse desarrollándolos y usando las propias ecuaciones de Cardano para el polinomio $p(x)$, es decir, usando que $\alpha+\beta+\gamma=0$, $\alpha\beta+\beta\gamma+\alpha\gamma=-3$ y $\alpha\beta\gamma=-1$.

Si crees que el enunciado contiene un error o imprecisión o bien crees que la información sobre la procedencia del problema es incorrecta, puedes notificarlo usando los siguientes botones:
Informar de error en enunciado Informar de procedencia del problema
Problema 278
Calcular $a$ y $b$ sabiendo que el polinomio \[p(x)=2x^3+ax^2+bx-16\] tiene una raíz doble y una raíz simple y que una de ellas es igual a 2.
pistasolución 1info
Pista. Expresa $p(x)=2(x-2)(x-\beta)^2$ ó $p(x)=2(x-\alpha)(x-2)^2$, desarrolla estos productos e iguala términos.
Solución. Que $p(x)$ tenga una raíz simple $\alpha$ y una doble $\beta$, nos dice que podemos escribir el polinomio como \[p(x)=2(x-\alpha)(x-\beta)^2{.}\] Ahora tendremos que distinguir dos casos, dependiendo de que $\alpha=2$ ó $\beta=2$.
  • Si $\alpha=2$, entonces desarrollamos \[p(x)=2(x-2)(x-\beta)^2=2x^3-4(\beta+1)x^2+2\beta(\beta+4)x-4\beta^2{.}\] Como el término independiente tiene que ser igual a $-16$, deducimos que $\beta^2=4$, es decir, $\beta=2$ ó $\beta=-2$. Obviamente $\beta=2$ tiene que descartarse ya que en tal caso $p(x)$ tendría una raíz triple, luego nos queda $\beta=-2$, en cuyo caso \[p(x)=2x^3-12x^2+24x-16,\] y por tanto $a=-12$ y $b=24$.
  • Si $\beta=2$, entonces podemos desarrollar \[p(x)=2(x-\alpha)(x-2)^2=2x^3-2(\alpha+4)x^2+8(\alpha-1)x+8\alpha{,}\] y el término independiente nos dice en este caso que $\alpha=2$, luego tenemos una raíz triple y tenemos que descartar este caso.
Deducimos que los únicos valores que cumplen el enunciado son $a=-12$ y $b=24$.
Si crees que el enunciado contiene un error o imprecisión o bien crees que la información sobre la procedencia del problema es incorrecta, puedes notificarlo usando los siguientes botones:
Informar de error en enunciado Informar de procedencia del problema
Problema 275
Determinar todos los pares de polinomios $p(x)$ y $q(x)$ con coeficientes reales tales que \[x^2q(x)=p(q(x)){.}\]
pistasolución 1info
Pista. Quitando los casos en que $q(x)$ es constante, demuestra que $p(x)=xr(x)$ para cierto polinomio $r(x)$ y que el producto de los grados de $r(x)$ y $q(x)$ es igual a $2$.
Solución. En primer lugar, si $q(x)=a$ es constante, llegamos a que $ax^2=p(a)$, de donde $a=0$. En tal caso $p(0)=0$ y, por tanto, existe otro polinomio $r(x)$ tal que $p(x)=xr(x)$. Es fácil ver que $p(x)=xr(x)$ y $q(x)=0$ cumplen la ecuación del enunciado, luego son las únicas soluciones en que $q(x)$ es constante.

Supongamos entonces que $q(x)$ no es constante, luego podemos tomar una raíz $\lambda$ de $q(x)$ (posiblemente $\lambda$ sea un número complejo). Sustituyendo $x=\lambda$ en la ecuación original, tenemos que $p(0)=0$ luego ha de existir un polinomio $r(x)$ tal que $p(x)=xr(x)$. La ecuación original queda $x^2q(x)=q(x)r(q(x))$ y, como $q(x)$ no es constante cero, podemos simplificar a $r(q(x))=x^2$. Ahora bien, el grado de la composición de dos polinomios es el producto de los grados y esto nos da dos posibilidades:

  • El grado de $r(x)$ es 2 y el grado de $q(x)$ es 1. En este caso $q(x)=mx+n$ para ciertos números reales $m,n\in\mathbb{R}$ y $m\neq 0$, luego el cambio de variable $x=\frac{y-n}{m}$ nos dice que \[\left(\frac{y-n}{m}\right)^2=x^2=r(q(x))=r\left(m\frac{y-n}{m}+n\right)=r(y),\] para cualquier $y$. Tenemos así las soluciones dadas por $q(x)=mx+n$ y $p(x)=xr(x)=x(\frac{x-n}{m})^2$ para $m,n\in\mathbb{R}$ con $m\neq 0$ (se comprueba que son soluciones).
  • El grado de $r(x)$ es 1 y el grado de $q(x)$ es 2. En este caso $r(x)=ax+b$ para ciertos números reales $a,b\in\mathbb{R}$ no nulos, luego $r(q(x))=aq(x)+b=x^2$, y de aquí despejamos $q(x)=\frac{x^2-b}{a}$. Por tanto, tenemos las soluciones dadas por $q(x)=\frac{x^2-b}{a}$ y $p(x)=xr(x)=ax^2+bx$ (que también se comprueba que son soluciones).

En resumen, toda solución de la ecuación cae en una de las siguientes familias:

  • $(p(x),q(x))=(xr(x),0)$ para cualquier polinomio $r(x)$ con coeficientes reales.
  • $(p(x),q(x))=(x(\frac{x-n}{m})^2,mx+n)$, para cualesquiera $m,n\in\mathbb{R}$ con $m\neq 0$.
  • $(p(x),q(x))=(ax^2+bx,\frac{x^2-b}{a})$ para cualesquiera $a,b\in\mathbb{R}$ con $a\neq 0$.

Si crees que el enunciado contiene un error o imprecisión o bien crees que la información sobre la procedencia del problema es incorrecta, puedes notificarlo usando los siguientes botones:
Informar de error en enunciado Informar de procedencia del problema
Problema 266
Supongamos que $P(x)$, $Q(x)$, $R(x)$ y $S(x)$ son polinomios tales que \[P(x^5)+xQ(x^5)+x^2R(x^5)=(1+x+x^2+x^3+x^4)S(x).\] Demostrar que $P(x)$ es divisible entre $x-1$.
pistasolución 1info
Pista. El hecho de que aparezcan $x^5$ y $1+x+x^2+x^3+x^4$, puede hacernos sospechar que usar raíces quintas es adecuado.
Solución. Demostraremos que $P(1)=0$, lo que nos dirá que $x-1$ es un factor de $P(x)$. La idea en este problema es apelar a la igualdad $(x-1)(1+x+x^2+x^3+x^4)=x^5-1$, que nos dice que $1+x+x^2+x^3+x^4$ tiene cuatro raíces complejas que son las raíces quintas de la unidad (salvo 1). En particular, las soluciones de $1+x+x^2+x^3+x^4$ se pueden expresar como $\{z,z^2,z^3,z^4\}$, siendo $z$ un número complejo distinto de 1 y tal que $z^5=1$.

Evaluando la igualdad del enunciado en $\{z,z^2,z^3,z^4\}$, obtenemos que \begin{eqnarray*} P(1)+zQ(1)+z^2R(1)&=&0,\\ P(1)+z^2Q(1)+z^4R(1)&=&0,\\ P(1)+z^3Q(1)+z^6R(1)&=&0,\\ P(1)+z^4Q(1)+z^8R(1)&=&0.\\ \end{eqnarray*} Esto puede verse como un sistema de cuatro ecuaciones lineales con tres incógnitas ($P(1)$, $Q(1)$ y $R(1)$). La matriz de coeficientes está dada por \[A=\left(\begin{matrix}1&z&z^2\\1&z^2&z^4\\1&z^3&z^6\\1&z^4&z^8\end{matrix}\right).\] Si eliminamos la última fila, el determinante de la matriz $3\times 3$ resultante es igual a $z^8-2z^7+2z^5-z^4$. Si usamos que $z^5=1$, podemos simplificarlo a $-z^4+z^3-2z^2+2$ y, usando que $z^4=-z^3-z^2-z-1$, podemos simplificarlo finalmente a $2z^3-z^2+z+3$. Si hacemos el mismo proceso eliminando la primera fila en lugar de la última, llegamos a que el determinante de la matriz $3\times 3$ resultante es $2z^3-z^2+z-2$. Evidentemente, los dos determinantes no pueden ser cero simultáneamente ya que se diferencian en 5 unidades, lo que nos dice que la matriz $A$ tiene rango 3 y, por tanto, el sistema de ecuaciones tiene como única solución la trivial $P(1)=Q(1)=R(1)=0$. Esto concluye la demostración.

Si crees que el enunciado contiene un error o imprecisión o bien crees que la información sobre la procedencia del problema es incorrecta, puedes notificarlo usando los siguientes botones:
Informar de error en enunciado Informar de procedencia del problema
José Miguel Manzano © 2010-2024. Esta página ha sido creada mediante software libre